LSAT and Law School Admissions Forum

Get expert LSAT preparation and law school admissions advice from PowerScore Test Preparation.

 jrc3813
  • Posts: 53
  • Joined: Apr 16, 2017
|
#34853
For this question I got B as the right answer but I wasn't 100% sure. This was a must be true so I knew the answer was going to be based on the facts and would be narrow in scope. Also because it was dealing with a possible causal link, the language would be soft. So that eliminated D and E right away and left A, B, and C.

A does say "tends to" so that's good, but it broadened the scope to include "exercise" rather than just "cycling." It also said "benefit" and the stimulus never really says anything about benefit although it implies it.

B was good because it mentions professional cyclists and "tends to depend at least in part."

And is C wrong because it says "best" and whenever superlatives or comparative terms appear in MBT questions you can usually eliminate them? Is that because it implies a strong causal link?

Basically I'm mostly sure B is right but A seems possible and I would like to be able to understand why C is wrong better.
User avatar
 Jonathan Evans
PowerScore Staff
  • PowerScore Staff
  • Posts: 726
  • Joined: Jun 09, 2016
|
#35011
Hi, JRC!

Good analysis. Well done eliminating (D) and (E). Since we're looking for the "most strongly supported" answer choice here, it is unsurprising that there are relative possible degrees of support among answer choices. You are correct that the superlative "best" in (C) makes it unattractive; there is not adequate support for this contention.

Now notice the distinction between (A) and (B). A gives an inverse correlation between heart rate and sustained exercise and psychological benefit. Based solely on the evidence, this contention is unsupported. For instance, it is unclear whether there might be a direct correlation between the two phenomena up to a certain point, i.e. it could be that there is increasing psychological benefit up to a certain heart rate, which subsequently drops off. Further, this answer departs from the evidence in the problem (cyclists) and makes a statement about exercise in general. Essentially, (A) is an answer that is meant to "sound" plausible but actually has little support.

The "depends in part" phrase in (B) as well as its limited scope make it the best option.

Good job!
User avatar
 queenbee
  • Posts: 75
  • Joined: Sep 18, 2022
|
#97484
hi
would you please help explain why C is incorrect? I didnt choose B because the lowest intensity was the only one of the three referenced that showed a benefit. I wasn't sure about C because it stated "no higher than 60%". Is that the reason why C is wrong?
Thank you
 Robert Carroll
PowerScore Staff
  • PowerScore Staff
  • Posts: 1787
  • Joined: Dec 06, 2013
|
#97544
queenbee,

As you said, only one of three referenced showed a benefit. So...what's the benefit of 65% intensity? 72%? I could go on. We're only told about three data points. We don't know where the maximum benefits are - we know the highest benefits among the three levels discussed, but not the highest benefits overall, because not every intensity level was tested. This is why answer choice (C) is wrong - how can we know 60% was the best, not just the best we know about?

The fact that the lowest intensity showed the largest benefits (at least, that we're aware of!) doesn't make answer choice (B) wrong. When one thing depends on another, that doesn't mean that an increase in one produces an increase in another - it means that changes in one will produce changes in the other. It's more general than "more of A, more of B" - it could mean "more of A, LESS of B", for instance. In the situation here, there does seem to be some connection between intensity and benefits. That's all required for answer choice (B) to be true, because it does say "at least in part", which is a pretty weak statement, and therefore requires less evidence. The facts in the stimulus seem to show some correlation, perhaps negative, between exercise intensity and health benefits. That's all we need for answer choice (B).

Robert Carroll

Get the most out of your LSAT Prep Plus subscription.

Analyze and track your performance with our Testing and Analytics Package.